문제지 PDF 파일을 로드하고 있습니다. 한참 걸릴 수도 있어요 ㅠㅠ 30초 이상 걸리면 새로고침을 한번 해보세요.

2023 7급 서울시 통신이론

1번

  1. FX()=0F_X(-\infty)=0

    이다.

  2. 그렇다. 모든 확률의 합이라고 생각하면 된다.

  3. 2번과 같다.

  4. 누적이기 때문에 FX(x)F_X(x)는 단조증가함수이므로 옳다.

따라서 답은 1번 이다.

2번

먼저 파스발의 정리를 이용해서 풀어보자. 주어진 신호를 복소 지수함수로 표현하면

x(t)=1j(ej2πtej2πt)+2(ej3πt+ej3πt)x(t)=\frac{1}{j}\left(e^{j2\pi t}-e^{-j2\pi t}\right)+2\left(e^{j3\pi t} + e^{-j3\pi t} \right)

이므로 x(t)x(t)의 푸리에 변환은

X(f)=1j(δ(f1)δ(f+1))+2(δ(f3)+δ(f+3))X(f)=\frac{1}{j}(\delta(f-1)-\delta(f+1))+2(\delta(f-3)+\delta(f+3))

이다. 따라서

SX(f)=1j2(δ(f1)+δ(f+1))+22(δ(f3)+δ(f+3))=δ(f1)+δ(f+1)+4δ(f3)+4δ(f+3)\begin{equation} \begin{split} S_X(f)&=\left\vert\frac{1}{j}\right\vert^2(\delta(f-1)+\delta(f+1))+2^2(\delta(f-3)+\delta(f+3))\\ &=\delta(f-1)+\delta(f+1)+4\delta(f-3)+4\delta(f+3) \end{split} \end{equation}

이므로 전력은

P=1+1+4+4=10P=1+1+4+4=10

이고 답은 3번 이다.

그러나 굳이 이렇게 할 필요는 없다. 그냥 시간 영역에서 전력을 구하면 되는데, 2sin(2πt)2sin(2\pi t)4cos(3πt)4cos(3\pi t)는 직교하므로(왜일까?) 각 성분의 전력들을 그냥 더하면 된다. 한편 acos(2πft)a cos(2\pi f t)의 전력은

11f01fa2cos2(2πft)dt=11f×a2×1201f(cos(4πft)+1)dt=a22(14πf(sin(4π)sin(0))+1)=a22\begin{equation} \begin{split} \frac{1}{\frac{1}{f}}\int_0 ^\frac{1}{f} a^2 cos^2(2\pi f t)dt&=\frac{1}{\frac{1}{f}}\times a^2\times \frac{1}{2}\int_0 ^\frac{1}{f}(cos(4\pi f t)+1)dt\\ &=\frac{a^2}{2}\left( \frac{1}{4\pi f}(sin(4\pi)-sin(0)) +1 \right)\\ &=\frac{a^2}{2} \end{split} \end{equation}

이고, asin(2πft)a sin(2\pi f t)도 마찬가지임에 따라

P=12(22+42)=10\begin{equation} \begin{split} P&=\frac{1}{2}(2^2+4^2)\\ &=10 \end{split} \end{equation}

이다.

3번

  1. FM이다. FM은 시간과 관계있는 주파수에 정보가 실리므로 시간 정보를 사용하지 않는 포락선 검파를 할 수 없고 미분해야 한다.
  2. SSB, 그 중에서도 SSB-SC이다. 포락선 검파를 위한 반송파 성분이 없으므로 포락선 검파가 불가하다.
  3. DSB-SC이다. 2번과 마찬가지이다.
  4. DSB-LC 또는 DSB-FC, DSB-TC라고도 한다. 반송파 성분이 있으므로 cos(2πfct)(x(t)+1)cos(2\pi f_c t)(x(t)+1)의 포락선인 x(t)+1x(t)+1을 이용해서 포락선 검파가 가능하다.

따라서 답은 4번 이다.

4번

해밍 부호는 선형 부호로 현재 값과 과거 값이 상관이 없다. 상관이 있는 것은 콘볼루션 코드(길쌈 부호)이다. 따라서 답은 3번 이다.

5번

Z=aiXiZ=\sum a_iX_i

일 때

Var(Z)=ai2Var(Xi)Var(Z)=\sum a_i^2 Var(X_i)

이다.(직접 증명해보자) 주어진 조건에서 ai=1,Var(Xi)=1a_i=1, Var(X_i)=1이므로

Var(Z)=1×50=50Var(Z)=1\times 50=50

이므로 답은 3번 이다.

6번

주어진 신호에 포함된 주파수 성분은 각각 2 Hz와 8 Hz이다. 샘플링 주파수가 10 Hz이므로 샘플링된 후의 주파수 성분들은 10k±2,10k±810k \pm 2, 10k \pm 8들이다. 즉 2,8,12,18,22,28,2, 8, 12, 18, 22, 28, \cdots Hz인데, 복원을 위해 LPF를 사용하면 추출되는 것은 2 Hz 성분 뿐이다. 따라서 답은 1번 이다.

7번

세 각주파수 중 중간의 각주파수가 3000π3000\pi rad/s이므로

A=4A=4

이다. 또한 이 각주파수를 중심으로 ±300π\pm 300\pi rad/s의 각주파수 편차가 있으므로

s(t)=2cos(300πt)s(t)=2cos(300\pi t)

임을 알 수 있다. 그래야

2cos(300πt)cos(3000πt)=cos((3000+300)πt)+cos((3000300)πt)2cos(300\pi t)cos(3000\pi t)=cos((3000+300)\pi t)+cos((3000-300)\pi t)

일 것이기 때문이다. 따라서 기저대역 신호 주파수는

fm=150 Hzf_m=150\text{ Hz}

이고 변조지수는

m=24=0.5m=\frac{2}{4}=0.5

이므로 답은 @번 이다.

8번

초록색 공이 꺼내질 확률은

P(X)P(초록색X)+P(Y)P(초록색Y)=0.6×30120+0.4×3280=0.6×14+0.4×0.4=0.15+0.16=0.31\begin{equation} \begin{split} P(X)P(\text{초록색}|X)+P(Y)P(\text{초록색}|Y)&=0.6\times\frac{30}{120}+0.4\times\frac{32}{80}\\ &=0.6\times\frac{1}{4}+0.4\times0.4\\ &=0.15+0.16\\ &=0.31 \end{split} \end{equation}

이다. 따라서 답은 4번 이다.

9번

L(y)=ln(12πe(y1)2212πe(y+1)22)=ln(e(y1)22+(y+1)22)=ln(e2y)=2y\begin{equation} \begin{split} L(y)&=ln\left(\frac{\frac{1}{\sqrt{2\pi}}e^{-\frac{(y-1)^2}{2}}} {\frac{1}{\sqrt{2\pi}}e^{-\frac{(y+1)^2}{2}}}\right)\\ &=ln\left( e^{-\frac{(y-1)^2}{2}+\frac{(y+1)^2}{2}} \right)\\ &=ln(e^{2y})\\ &=2y \end{split} \end{equation}

이다. 따라서

L(0.5)=2×0.5=1L(0.5)=2\times 0.5=1

이므로 답은 3번 이다.

10번

  1. (7,4) 부호의 경우 dmin=3d_{min}=3이므로 31=23-1=2개의 비트 오류를 검출할 수 있다.
  2. 그렇다.
  3. [111100011001101010101]×[0010100]=[110]\begin{equation} \begin{split} \begin{bmatrix} 1&1&1&1&0&0&0\\ 1&1&0&0&1&1&0\\ 1&0&1&0&1&0&1 \end{bmatrix} \times \begin{bmatrix} 0\\0\\1\\0\\1\\0\\0 \end{bmatrix}&= \begin{bmatrix} 1\\1\\0 \end{bmatrix} \end{split} \end{equation}
    이므로 패리티 검사 행렬의 두 번쨰 열과 같다. 따라서 두 번째 비트에서 오류가 발생하였을 것이므로 정정된 비트열은
    [0110100]\begin{bmatrix} 0&1&1&0&1&0&0 \end{bmatrix}
    이다.
  4. [111100011001101010101]×[1010000]=[010]\begin{equation} \begin{split} \begin{bmatrix} 1&1&1&1&0&0&0\\ 1&1&0&0&1&1&0\\ 1&0&1&0&1&0&1 \end{bmatrix} \times \begin{bmatrix} 1\\0\\1\\0\\0\\0\\0 \end{bmatrix}&= \begin{bmatrix} 0\\1\\0 \end{bmatrix} \end{split} \end{equation}
    이므로 결과가 영행렬이 아니다. 따라서 주어진 비트열은 G4×7G_{4\times7}의 한 행이 될 수 없다.

따라서 답은 4번 이다.

11번

한 개의 부반송파 대역폭은

5.12×106256=2×104 Hz\frac{5.12\times 10^6}{256}=2\times 10^4\text{ Hz}

이다. 한편 QPSK를 이용함에 따라 한 부반송파당 2개의 비트가 실린다. 따라서 초당 비트 전송률은

2×104×2×256 bps2\times 10^4\times 2\times 256\text{ bps}

이다. 그러므로 스펙트럼 효율을 구하면

2×104×2×256×15.12×106=2 bps/Hz2\times 10^4\times2\times 256\times \frac{1}{5.12\times 10^6}=2\text{ bps/Hz}

이고 답은 2번 이다.

12번

  1. RXY(τ)=E[(X(t))Y(t+τ)]=E[X(tτ)Y(t)]=E[Y(t)X(tτ)]=RYX(τ)\begin{equation} \begin{split} R_{XY}(\tau)&=E[(X(t))Y(t+\tau)]\\ &=E[X(t-\tau)Y(t)]\\ &=E[Y(t)X(t-\tau)]\\ &=R_{YX}(-\tau) \end{split} \end{equation}

    이므로 옳지 않다.

  2. 산술-기하 평균 관계

    2aba+b4aba2+2ab+b2ab12(a2+b2)2\sqrt{ab}\leq a+b \Rightarrow 4ab\leq a^2+2ab+b^2 \Rightarrow ab \leq \frac{1}{2}(a^2+b^2)

    을 이용하자.

    RXY(τ)=E[X(t)Y(t+τ)]E[12(X2(t)+Y2(t+τ))]=12E[X2(t)]+E[Y2(t+τ)]=12[RX(0)+RY(0)]\begin{equation} \begin{split} |R_{XY}(\tau)|&=|E[X(t)Y(t+\tau)]|\\ &\leq \left|E\left[ \frac{1}{2}(X^2(t)+Y^2(t+\tau)) \right]\right|\\ &=\frac{1}{2}\left|E[X^2(t)]+E[Y^2(t+\tau)] \right|\\ &=\frac{1}{2}[R_X(0)+R_Y(0)] \end{split} \end{equation}

    이므로 옳다.

  3. 2번의 관계식에서 τ=0\tau=0을 대입해보면 이 등식은 성립하지 않음을 알 수 있다.

  4. 성립할 이유가 없다.

따라서 답은 2번 이다.

13번

나이퀴스트율은 최고 주파수의 2배인 8,000 Hz이다. 따라서 이의 1.5배 주파수는 12,000 Hz이다. 한편 롤오프율이 α=0.4\alpha=0.4이므로 대역폭은

1+0.42×12000=8400 Hz\frac{1+0.4}{2}\times 12000=8400 \text{ Hz}

가 된다. 그러므로 답은 3번 이다.

14번

허프만 코딩을 직접 해보자. 평균 길이를 구해보면

H=1×610+2×940+3×110+4×120+4×140=1.65\begin{equation} \begin{split} H&=1\times \frac{6}{10}+2\times \frac{9}{40}+3\times \frac{1}{10}+4\times \frac{1}{20}+4\times \frac{1}{40}\\ &=1.65 \end{split} \end{equation}

이므로 답은 2번 이다.

15번

  1. 절반은 +1, 나머지 절반은 -1을 전송하므로 대역폭은 RZ와 같다.
  2. 그렇다.
  3. 그렇다.
  4. 그렇다. 중간의 상태 변화 지점들을 클럭으로 사용할 수 있다.

따라서 답은 1번 이다.

16번

채널 용량은

C=Wlog2(1+SNR)=500log2(1+15)=2 Mbps\begin{equation} \begin{split} C&=W log_2(1+SNR)\\ &=500 log_2(1+15)\\ &=2\text{ Mbps} \end{split} \end{equation}

이므로 답은 4번 이다.

17번

식을 세워서 정리한 다음 라플라스 변환하면

(x(t)y(t))dt+e(t)=y(t)Y(s)(1+1s)=1sX(s)+E(s)Y(s)=1s1+1sX(s)+11+1sE(s)=11+sX(s)+s1+sE(s)\begin{equation} \begin{split} \int (x(t)-y(t))dt+e(t)&=y(t)\\ \Rightarrow Y(s)\left(1+\frac{1}{s}\right)&=\frac{1}{s}X(s)+E(s)\\ \Rightarrow Y(s)&=\frac{\frac{1}{s}}{1+\frac{1}{s}}X(s)+\frac{1}{1+\frac{1}{s}}E(s)\\ &=\frac{1}{1+s}X(s)+\frac{s}{1+s}E(s) \end{split} \end{equation}

이므로 답은 1번 이다.

18번

정합 필터를 쓰면 수신 신호를 통과시킨 후 샘플링 시의 SNR이 최대가 된다. 따라서 답은 4번 이다.

19번

VLF나 SHF의 대역폭을 일일이 외울 수도 있겠으나, 10310^3 배 단위로 대역 이름이 달라짐을 생각한다면 10의 지수승이 3의 배수인 것을 고르면 된다. 주어진 보기 중에서는 3번 밖에 없다.

20번

나이퀴스트율은 차단주파수의 2배인 8 kHz이다. 따라서 이의 1.5배는 12 kHz이다. 각 표본을 8비트로 변환하여 전송하면 전송속도는

12×8=96 kbps12\times 8=96\text{ kbps}

가 되므로 답은 4번 이다.